The Student Room Group

Spherical coordinates

Calculate the z-component of the centre of mass for a northern hemisphere of radius RR with constant density ρ0>0\rho_0 > 0 using spherical coordinates (r,θ,φ)(r,\theta, \varphi ) defined by:

x(r,θ,φ)=rsinθcosφ            0r<x(r,\theta, \varphi) = r\sin\theta\cos\varphi \;\;\;\;\;\;0 \leq r < \infty
y(r,θ,φ)=rsinθsinφ            0θπy(r,\theta, \varphi) = r\sin\theta\sin\varphi \;\;\;\;\;\;\, 0 \leq \theta \leq \pi
z(r,θ,φ)=rcosθ                            0φ<2πz(r,\theta, \varphi) = r\cos\theta \;\;\;\;\;\;\;\;\;\;\;\;\;\; 0 \leq \varphi < 2\pi

For a solid with density ρ(r)\rho (\bf{r}) occupying a region R\cal{R},

zcm=1MRzρ(r)  dV z_{cm} = \frac{1}{M} \iiint_{\cal{R}} z \rho ({\bf r})\;dV where M=Rρ(r)  dVM= \iiint_{\cal{R}} \rho ({\bf r})\;dV

I have the solution but I'm wondering why the limits of θ\theta is [0,π/2][0,\pi /2] for calculating M then it changes to [0,π][0,\pi] when calculating zcmz_{cm} ?

M=Rρ0  dV=0Rdr0π2dθ02πdφ  r2sinθ=2π3R3ρ0M = \iiint _{\cal{R}} \rho_0\;dV = \int_0^R dr \int_0^{\frac{\pi}{2}} d\theta \int_0^{2\pi} d\varphi \; r^2\sin\theta = \frac{2\pi}{3}R^3 \rho_0.

Mzcm=Rzρ0  dV=0Rdr0πdθ02πdφ  rcosθr2sinθ=M38RMz_{cm} = \iiint _{\cal{R}} z \rho_0\;dV = \int_0^R dr \int_0^{\pi} d\theta \int_0^{2\pi} d\varphi \; r\cos\theta r^2\sin\theta = M \frac{3}{8} R.
(edited 13 years ago)
Reply 1
I'm guessing that it's a mistake; it doesn't look like they've done anything other than multiply the integrand by zz, so it shouldn't affect the limits in any way.
Reply 2
Original post by nuodai
I'm guessing that it's a mistake; it doesn't look like they've done anything other than multiply the integrand by zz, so it shouldn't affect the limits in any way.


So I'm guessing both integrals should have θ\theta limits [0,π][0,\pi] for a 'northern hemisphere'?
Reply 3
Original post by TheEd
So I'm guessing both integrals should have θ\theta limits [0,π][0,\pi] for a 'northern hemisphere'?


No, they should both be [0,π2][0, \frac{\pi}{2}]. If P is a point in R3\mathbb{R}^3 then θ\theta tells you the angle that OP makes with the positive z-axis, so if it went up to π\pi then you'd have a full sphere.
Reply 4
Original post by nuodai
No, they should both be [0,π2][0, \frac{\pi}{2}]. If P is a point in R3\mathbb{R}^3 then θ\theta tells you the angle that OP makes with the positive z-axis, so if it went up to π\pi then you'd have a full sphere.


Yeah makes sense now. Ta.

Quick Reply

Latest